About lattice of finitely generated projective module The Next CEO of Stack OverflowAbout Inner product spaceAbout symmetric inner product spaceCan we contruct a basis in a finitely generated moduleBasis of a subset of finitely generated torsion free moduleProve module $M$ is finitely generated if $N$ and $M/N$ are finitely generatedProof about finitely generated torsion-free R-module M is free, where R is a PID(Zelmanowitz) Regular Module but not ProjectiveFinitely generated module over an orderprojective module which is a submodule of a finitely generated free moduleWhat can be said of the lattices of submodules of a noetherian/finitely generated module?Rings over which torsion free module is projectiveFinitely generated free module is projective.

How easy is it to start Magic from scratch?

How do spells that require an ability check vs. the caster's spell save DC work?

What does "Its cash flow is deeply negative" mean?

Text adventure game code

Visit to the USA with ESTA approved before trip to Iran

Apart from "berlinern", do any other German dialects have a corresponding verb?

Should I tutor a student who I know has cheated on their homework?

Go Pregnant or Go Home

How to count occurrences of text in a file?

What is the purpose of the Evocation wizard's Potent Cantrip feature?

Why didn't Khan get resurrected in the Genesis Explosion?

How does practicing restraint and performing actions of merit purify the mind?

How did people program for Consoles with multiple CPUs?

Why doesn't a table tennis ball float on the surface? How do we calculate buoyancy here?

What is the difference between "behavior" and "behaviour"?

If I blow insulation everywhere in my attic except the door trap, will heat escape through it?

How do scammers retract money, while you can’t?

Which organization defines CJK Unified Ideographs?

Is it safe to use c_str() on a temporary string?

What can we do to stop prior company from asking us questions?

How do I go from 300 unfinished/half written blog posts, to published posts?

Only print output after finding pattern

How can I open an app using Terminal?

How should I support this large drywall patch?



About lattice of finitely generated projective module



The Next CEO of Stack OverflowAbout Inner product spaceAbout symmetric inner product spaceCan we contruct a basis in a finitely generated moduleBasis of a subset of finitely generated torsion free moduleProve module $M$ is finitely generated if $N$ and $M/N$ are finitely generatedProof about finitely generated torsion-free R-module M is free, where R is a PID(Zelmanowitz) Regular Module but not ProjectiveFinitely generated module over an orderprojective module which is a submodule of a finitely generated free moduleWhat can be said of the lattices of submodules of a noetherian/finitely generated module?Rings over which torsion free module is projectiveFinitely generated free module is projective.










1












$begingroup$


Let $A$ be euclidian ring and $K$ be its field of a fraction.Let $(V,B)$ be a nonzero IPS (inner product space) over $K$. A finitely generated A-submodule
$L ⊆ V $is said to be an $A$ -lattice in $V$ if $L$ contains a $K$-basis of $V$ . As we have
already observed, $L$ must be $A$-free since it is torsion-free, and it is easy to verify that
an $A$-basis for $L$ will automatically be a $K$-basis for $V$.



Attempt: I know that $V$ is finitely generated projective module since $(V, B)$ is inner product space and $B$ is regular bilinear form. Now question is what is the meaning of $L$ contains a $K$-basis of $V$ and how to verify $A$-basis for L will automatically be a $K$-basis for $V$ ; it certainly means we can extend from basis of $L$ to basis of $V$?










share|cite|improve this question









$endgroup$
















    1












    $begingroup$


    Let $A$ be euclidian ring and $K$ be its field of a fraction.Let $(V,B)$ be a nonzero IPS (inner product space) over $K$. A finitely generated A-submodule
    $L ⊆ V $is said to be an $A$ -lattice in $V$ if $L$ contains a $K$-basis of $V$ . As we have
    already observed, $L$ must be $A$-free since it is torsion-free, and it is easy to verify that
    an $A$-basis for $L$ will automatically be a $K$-basis for $V$.



    Attempt: I know that $V$ is finitely generated projective module since $(V, B)$ is inner product space and $B$ is regular bilinear form. Now question is what is the meaning of $L$ contains a $K$-basis of $V$ and how to verify $A$-basis for L will automatically be a $K$-basis for $V$ ; it certainly means we can extend from basis of $L$ to basis of $V$?










    share|cite|improve this question









    $endgroup$














      1












      1








      1





      $begingroup$


      Let $A$ be euclidian ring and $K$ be its field of a fraction.Let $(V,B)$ be a nonzero IPS (inner product space) over $K$. A finitely generated A-submodule
      $L ⊆ V $is said to be an $A$ -lattice in $V$ if $L$ contains a $K$-basis of $V$ . As we have
      already observed, $L$ must be $A$-free since it is torsion-free, and it is easy to verify that
      an $A$-basis for $L$ will automatically be a $K$-basis for $V$.



      Attempt: I know that $V$ is finitely generated projective module since $(V, B)$ is inner product space and $B$ is regular bilinear form. Now question is what is the meaning of $L$ contains a $K$-basis of $V$ and how to verify $A$-basis for L will automatically be a $K$-basis for $V$ ; it certainly means we can extend from basis of $L$ to basis of $V$?










      share|cite|improve this question









      $endgroup$




      Let $A$ be euclidian ring and $K$ be its field of a fraction.Let $(V,B)$ be a nonzero IPS (inner product space) over $K$. A finitely generated A-submodule
      $L ⊆ V $is said to be an $A$ -lattice in $V$ if $L$ contains a $K$-basis of $V$ . As we have
      already observed, $L$ must be $A$-free since it is torsion-free, and it is easy to verify that
      an $A$-basis for $L$ will automatically be a $K$-basis for $V$.



      Attempt: I know that $V$ is finitely generated projective module since $(V, B)$ is inner product space and $B$ is regular bilinear form. Now question is what is the meaning of $L$ contains a $K$-basis of $V$ and how to verify $A$-basis for L will automatically be a $K$-basis for $V$ ; it certainly means we can extend from basis of $L$ to basis of $V$?







      abstract-algebra commutative-algebra modules projective-module






      share|cite|improve this question













      share|cite|improve this question











      share|cite|improve this question




      share|cite|improve this question










      asked yesterday









      maths studentmaths student

      6321521




      6321521




















          1 Answer
          1






          active

          oldest

          votes


















          0












          $begingroup$

          The meaning of $L$ containing a $K$-basis for $V$ is that there are $ell_1,ldots,ell_nin Lsubseteq V$ forming a basis for $V$ as a $K$-vector space (where $n=dim_K V$).



          Now suppose that we have $l_1,ldots,l_rin L$ are a basis for $L$ as a free $A$-module. (Note the different style of $l$). We want to prove that $r=n$ and that the elements $l_i$ are linearly independent over $K$, so that they form a $K$-basis.



          If $$sum_i=1^r c_il_i=0,$$
          with $c_iin K$, we can multiply by a common denominator $dne 0in A$, so that $c_idin A$. Then we get
          $$sum_i=1^r (c_id)l_i=0,$$
          but the $l_i$ were an $A$-basis for $L$, so $c_id=0$ for all $i$, and since $dne 0$, $c_i=0$ for all $i$.



          As for why $n=r$, observe that $V=KL=Kotimes_A L=Kotimes_A A^r = K^r$, so $r=dim K^r=dim V =n$.






          share|cite|improve this answer









          $endgroup$












          • $begingroup$
            @mathsstudent I don't usually participate in chat, if you have another question, feel free to ask it, and drop me a comment linking to it and I'll check it out.
            $endgroup$
            – jgon
            yesterday










          • $begingroup$
            @mathsstudent Yes
            $endgroup$
            – jgon
            yesterday










          • $begingroup$
            I ask doubt below sir.
            $endgroup$
            – maths student
            yesterday











          • $begingroup$
            math.stackexchange.com/questions/3164762/… I have ask it?
            $endgroup$
            – maths student
            yesterday










          • $begingroup$
            I have got corollary 3.9 can you please explain to me how to prove note below corollary 3.9.
            $endgroup$
            – maths student
            yesterday











          Your Answer





          StackExchange.ifUsing("editor", function ()
          return StackExchange.using("mathjaxEditing", function ()
          StackExchange.MarkdownEditor.creationCallbacks.add(function (editor, postfix)
          StackExchange.mathjaxEditing.prepareWmdForMathJax(editor, postfix, [["$", "$"], ["\\(","\\)"]]);
          );
          );
          , "mathjax-editing");

          StackExchange.ready(function()
          var channelOptions =
          tags: "".split(" "),
          id: "69"
          ;
          initTagRenderer("".split(" "), "".split(" "), channelOptions);

          StackExchange.using("externalEditor", function()
          // Have to fire editor after snippets, if snippets enabled
          if (StackExchange.settings.snippets.snippetsEnabled)
          StackExchange.using("snippets", function()
          createEditor();
          );

          else
          createEditor();

          );

          function createEditor()
          StackExchange.prepareEditor(
          heartbeatType: 'answer',
          autoActivateHeartbeat: false,
          convertImagesToLinks: true,
          noModals: true,
          showLowRepImageUploadWarning: true,
          reputationToPostImages: 10,
          bindNavPrevention: true,
          postfix: "",
          imageUploader:
          brandingHtml: "Powered by u003ca class="icon-imgur-white" href="https://imgur.com/"u003eu003c/au003e",
          contentPolicyHtml: "User contributions licensed under u003ca href="https://creativecommons.org/licenses/by-sa/3.0/"u003ecc by-sa 3.0 with attribution requiredu003c/au003e u003ca href="https://stackoverflow.com/legal/content-policy"u003e(content policy)u003c/au003e",
          allowUrls: true
          ,
          noCode: true, onDemand: true,
          discardSelector: ".discard-answer"
          ,immediatelyShowMarkdownHelp:true
          );



          );













          draft saved

          draft discarded


















          StackExchange.ready(
          function ()
          StackExchange.openid.initPostLogin('.new-post-login', 'https%3a%2f%2fmath.stackexchange.com%2fquestions%2f3164399%2fabout-lattice-of-finitely-generated-projective-module%23new-answer', 'question_page');

          );

          Post as a guest















          Required, but never shown

























          1 Answer
          1






          active

          oldest

          votes








          1 Answer
          1






          active

          oldest

          votes









          active

          oldest

          votes






          active

          oldest

          votes









          0












          $begingroup$

          The meaning of $L$ containing a $K$-basis for $V$ is that there are $ell_1,ldots,ell_nin Lsubseteq V$ forming a basis for $V$ as a $K$-vector space (where $n=dim_K V$).



          Now suppose that we have $l_1,ldots,l_rin L$ are a basis for $L$ as a free $A$-module. (Note the different style of $l$). We want to prove that $r=n$ and that the elements $l_i$ are linearly independent over $K$, so that they form a $K$-basis.



          If $$sum_i=1^r c_il_i=0,$$
          with $c_iin K$, we can multiply by a common denominator $dne 0in A$, so that $c_idin A$. Then we get
          $$sum_i=1^r (c_id)l_i=0,$$
          but the $l_i$ were an $A$-basis for $L$, so $c_id=0$ for all $i$, and since $dne 0$, $c_i=0$ for all $i$.



          As for why $n=r$, observe that $V=KL=Kotimes_A L=Kotimes_A A^r = K^r$, so $r=dim K^r=dim V =n$.






          share|cite|improve this answer









          $endgroup$












          • $begingroup$
            @mathsstudent I don't usually participate in chat, if you have another question, feel free to ask it, and drop me a comment linking to it and I'll check it out.
            $endgroup$
            – jgon
            yesterday










          • $begingroup$
            @mathsstudent Yes
            $endgroup$
            – jgon
            yesterday










          • $begingroup$
            I ask doubt below sir.
            $endgroup$
            – maths student
            yesterday











          • $begingroup$
            math.stackexchange.com/questions/3164762/… I have ask it?
            $endgroup$
            – maths student
            yesterday










          • $begingroup$
            I have got corollary 3.9 can you please explain to me how to prove note below corollary 3.9.
            $endgroup$
            – maths student
            yesterday















          0












          $begingroup$

          The meaning of $L$ containing a $K$-basis for $V$ is that there are $ell_1,ldots,ell_nin Lsubseteq V$ forming a basis for $V$ as a $K$-vector space (where $n=dim_K V$).



          Now suppose that we have $l_1,ldots,l_rin L$ are a basis for $L$ as a free $A$-module. (Note the different style of $l$). We want to prove that $r=n$ and that the elements $l_i$ are linearly independent over $K$, so that they form a $K$-basis.



          If $$sum_i=1^r c_il_i=0,$$
          with $c_iin K$, we can multiply by a common denominator $dne 0in A$, so that $c_idin A$. Then we get
          $$sum_i=1^r (c_id)l_i=0,$$
          but the $l_i$ were an $A$-basis for $L$, so $c_id=0$ for all $i$, and since $dne 0$, $c_i=0$ for all $i$.



          As for why $n=r$, observe that $V=KL=Kotimes_A L=Kotimes_A A^r = K^r$, so $r=dim K^r=dim V =n$.






          share|cite|improve this answer









          $endgroup$












          • $begingroup$
            @mathsstudent I don't usually participate in chat, if you have another question, feel free to ask it, and drop me a comment linking to it and I'll check it out.
            $endgroup$
            – jgon
            yesterday










          • $begingroup$
            @mathsstudent Yes
            $endgroup$
            – jgon
            yesterday










          • $begingroup$
            I ask doubt below sir.
            $endgroup$
            – maths student
            yesterday











          • $begingroup$
            math.stackexchange.com/questions/3164762/… I have ask it?
            $endgroup$
            – maths student
            yesterday










          • $begingroup$
            I have got corollary 3.9 can you please explain to me how to prove note below corollary 3.9.
            $endgroup$
            – maths student
            yesterday













          0












          0








          0





          $begingroup$

          The meaning of $L$ containing a $K$-basis for $V$ is that there are $ell_1,ldots,ell_nin Lsubseteq V$ forming a basis for $V$ as a $K$-vector space (where $n=dim_K V$).



          Now suppose that we have $l_1,ldots,l_rin L$ are a basis for $L$ as a free $A$-module. (Note the different style of $l$). We want to prove that $r=n$ and that the elements $l_i$ are linearly independent over $K$, so that they form a $K$-basis.



          If $$sum_i=1^r c_il_i=0,$$
          with $c_iin K$, we can multiply by a common denominator $dne 0in A$, so that $c_idin A$. Then we get
          $$sum_i=1^r (c_id)l_i=0,$$
          but the $l_i$ were an $A$-basis for $L$, so $c_id=0$ for all $i$, and since $dne 0$, $c_i=0$ for all $i$.



          As for why $n=r$, observe that $V=KL=Kotimes_A L=Kotimes_A A^r = K^r$, so $r=dim K^r=dim V =n$.






          share|cite|improve this answer









          $endgroup$



          The meaning of $L$ containing a $K$-basis for $V$ is that there are $ell_1,ldots,ell_nin Lsubseteq V$ forming a basis for $V$ as a $K$-vector space (where $n=dim_K V$).



          Now suppose that we have $l_1,ldots,l_rin L$ are a basis for $L$ as a free $A$-module. (Note the different style of $l$). We want to prove that $r=n$ and that the elements $l_i$ are linearly independent over $K$, so that they form a $K$-basis.



          If $$sum_i=1^r c_il_i=0,$$
          with $c_iin K$, we can multiply by a common denominator $dne 0in A$, so that $c_idin A$. Then we get
          $$sum_i=1^r (c_id)l_i=0,$$
          but the $l_i$ were an $A$-basis for $L$, so $c_id=0$ for all $i$, and since $dne 0$, $c_i=0$ for all $i$.



          As for why $n=r$, observe that $V=KL=Kotimes_A L=Kotimes_A A^r = K^r$, so $r=dim K^r=dim V =n$.







          share|cite|improve this answer












          share|cite|improve this answer



          share|cite|improve this answer










          answered yesterday









          jgonjgon

          16.1k32143




          16.1k32143











          • $begingroup$
            @mathsstudent I don't usually participate in chat, if you have another question, feel free to ask it, and drop me a comment linking to it and I'll check it out.
            $endgroup$
            – jgon
            yesterday










          • $begingroup$
            @mathsstudent Yes
            $endgroup$
            – jgon
            yesterday










          • $begingroup$
            I ask doubt below sir.
            $endgroup$
            – maths student
            yesterday











          • $begingroup$
            math.stackexchange.com/questions/3164762/… I have ask it?
            $endgroup$
            – maths student
            yesterday










          • $begingroup$
            I have got corollary 3.9 can you please explain to me how to prove note below corollary 3.9.
            $endgroup$
            – maths student
            yesterday
















          • $begingroup$
            @mathsstudent I don't usually participate in chat, if you have another question, feel free to ask it, and drop me a comment linking to it and I'll check it out.
            $endgroup$
            – jgon
            yesterday










          • $begingroup$
            @mathsstudent Yes
            $endgroup$
            – jgon
            yesterday










          • $begingroup$
            I ask doubt below sir.
            $endgroup$
            – maths student
            yesterday











          • $begingroup$
            math.stackexchange.com/questions/3164762/… I have ask it?
            $endgroup$
            – maths student
            yesterday










          • $begingroup$
            I have got corollary 3.9 can you please explain to me how to prove note below corollary 3.9.
            $endgroup$
            – maths student
            yesterday















          $begingroup$
          @mathsstudent I don't usually participate in chat, if you have another question, feel free to ask it, and drop me a comment linking to it and I'll check it out.
          $endgroup$
          – jgon
          yesterday




          $begingroup$
          @mathsstudent I don't usually participate in chat, if you have another question, feel free to ask it, and drop me a comment linking to it and I'll check it out.
          $endgroup$
          – jgon
          yesterday












          $begingroup$
          @mathsstudent Yes
          $endgroup$
          – jgon
          yesterday




          $begingroup$
          @mathsstudent Yes
          $endgroup$
          – jgon
          yesterday












          $begingroup$
          I ask doubt below sir.
          $endgroup$
          – maths student
          yesterday





          $begingroup$
          I ask doubt below sir.
          $endgroup$
          – maths student
          yesterday













          $begingroup$
          math.stackexchange.com/questions/3164762/… I have ask it?
          $endgroup$
          – maths student
          yesterday




          $begingroup$
          math.stackexchange.com/questions/3164762/… I have ask it?
          $endgroup$
          – maths student
          yesterday












          $begingroup$
          I have got corollary 3.9 can you please explain to me how to prove note below corollary 3.9.
          $endgroup$
          – maths student
          yesterday




          $begingroup$
          I have got corollary 3.9 can you please explain to me how to prove note below corollary 3.9.
          $endgroup$
          – maths student
          yesterday

















          draft saved

          draft discarded
















































          Thanks for contributing an answer to Mathematics Stack Exchange!


          • Please be sure to answer the question. Provide details and share your research!

          But avoid


          • Asking for help, clarification, or responding to other answers.

          • Making statements based on opinion; back them up with references or personal experience.

          Use MathJax to format equations. MathJax reference.


          To learn more, see our tips on writing great answers.




          draft saved


          draft discarded














          StackExchange.ready(
          function ()
          StackExchange.openid.initPostLogin('.new-post-login', 'https%3a%2f%2fmath.stackexchange.com%2fquestions%2f3164399%2fabout-lattice-of-finitely-generated-projective-module%23new-answer', 'question_page');

          );

          Post as a guest















          Required, but never shown





















































          Required, but never shown














          Required, but never shown












          Required, but never shown







          Required, but never shown

































          Required, but never shown














          Required, but never shown












          Required, but never shown







          Required, but never shown







          Popular posts from this blog

          Triangular numbers and gcdProving sum of a set is $0 pmod n$ if $n$ is odd, or $fracn2 pmod n$ if $n$ is even?Is greatest common divisor of two numbers really their smallest linear combination?GCD, LCM RelationshipProve a set of nonnegative integers with greatest common divisor 1 and closed under addition has all but finite many nonnegative integers.all pairs of a and b in an equation containing gcdTriangular Numbers Modulo $k$ - Hit All Values?Understanding the Existence and Uniqueness of the GCDGCD and LCM with logical symbolsThe greatest common divisor of two positive integers less than 100 is equal to 3. Their least common multiple is twelve times one of the integers.Suppose that for all integers $x$, $x|a$ and $x|b$ if and only if $x|c$. Then $c = gcd(a,b)$Which is the gcd of 2 numbers which are multiplied and the result is 600000?

          Ingelân Ynhâld Etymology | Geografy | Skiednis | Polityk en bestjoer | Ekonomy | Demografy | Kultuer | Klimaat | Sjoch ek | Keppelings om utens | Boarnen, noaten en referinsjes Navigaasjemenuwww.gov.ukOffisjele webside fan it regear fan it Feriene KeninkrykOffisjele webside fan it Britske FerkearsburoNederlânsktalige ynformaasje fan it Britske FerkearsburoOffisjele webside fan English Heritage, de organisaasje dy't him ynset foar it behâld fan it Ingelske kultuergoedYnwennertallen fan alle Britske stêden út 'e folkstelling fan 2011Notes en References, op dizze sideEngland

          Հադիս Բովանդակություն Անվանում և նշանակություն | Դասակարգում | Աղբյուրներ | Նավարկման ցանկ